Board logo

标题: . 请教LSAT中一种逻辑的问法 [打印本页]

作者: jojofgasw    时间: 2004-7-7 10:54     标题: . 请教LSAT中一种逻辑的问法

Which one of the following, if true, most calls into question the XXX’argument?fficeffice" />

我理解成削弱,结果答案总是做错,搞不清楚到底是加强还是削弱XXX的观点。

For example:

The gray squirrel, introduced into local woodlands ten years ago, threatens the indigenous population of an endangered owl species, because the squirrels’ habitual stripping of tree bark destroys the trees in which the owls nest. Some local officials have advocated setting out poison for the gray squirrels. The officials argue that this measure, while eliminating the squirrels, would pose no threat to the owl population, since the poison would be placed in containers accessible only to squirrels and other rodents.

Which one of the following, if true, most calls into question the officials’ argument?

(A) One of the species whose members are likely to eat the poison is the red squirrel, a species on which owls do not prey.

(B) The owls whose nesting sites are currently being destroyed by the gray squirrels feed primarily on rodents.

(C) No indigenous population of any other bind species apart from the endangered owls is threatened by the gray squirrels.

(D) The owls that tare threatened build their nests in the tops of trees, but the gray squirrels strip away back from the trunks. (E) The officials’ plan entails adding the poison to food sources that are usually eaten by rodents but not by other animals.


作者: himba    时间: 2004-7-7 11:00

削弱fficeffice" />




欢迎光临 国际顶尖MBA申请交流平台--TOPWAY MBA (http://forum.topway.org/) Powered by Discuz! 7.2